Chapter 51: Assessment/ management diabetes questions, chapter 48 diabetes, Diabetes mellitus, Diabetes Medication2

¡Supera tus tareas y exámenes ahora con Quizwiz!

When the Type 1 diabetic patient asks why his 7 AM insulin has been changed from NPH insulin to 70/30 premixed insulin, the nurse explains that 70/30 insulin: a. is absorbed more rapidly into the bloodstream. b. has no peak action time and lasts all day. c. makes insulin administration easier and safer. d. give a bolus of rapid-acting insulin to prevent hyperglycemia after breakfast. the morning meal.

ANS: D 70/30 insulin is 30% rapid-acting and 70% intermediate-acting insulin. The rapid action of the 7 AM premixed insulin prevents hyperglycemia after the morning meal.

Which of the following regimens offers the best blood glucose control for persons with type 1 diabetes? a. A single anti-diabetes drugs b. Once daily insulin injections c. A combination of oral anti-diabetic medications d. Three or four injections per day of different types of insulin.

ANS: D Because persons with type 1 diabetes do not produce insulin, they require insulin and cannot be treated with oral anti-diabetic drugs. Several injections of insulin per day, calibrated to respond to measured blood glucose levels, offer the best blood glucose control and may prevent or postpone the retinal, renal, and neurological complications of diabetes.

A client is learning to inject insulin. Which action is important for the nurse to teach the client? a. "Do not use needles more than twice before discarding." b. "Massage the site for 1 full minute after injection." c. "Try to make the injection deep enough to enter muscle." d. "Keep the vial you are using in the pantry or the bedroom drawer."

ANS: D Cold insulin directly from the refrigerator is the most common cause of irritation (not infection) at the insulin injection site. Insulin in active use can be stored at room temperature. However, the bathroom is not the best place to store any medication because of increased heat and humidity. Needles should be used only once. Massage will not prevent or treat irritation from cold insulin. Insulin is given by subcutaneous, not intramuscular, injection.

A client with diabetes receives 10 units of regular insulin at 6:00 am and does not eat breakfast. About noon, what observation would the nurse expect to see? a. Polydipsia b. Polyphagia c. Polyuria d. Diaphoresis

ANS: D The nurse would expect symptoms of hypoglycemia, which include diaphoresis, shakiness, fatigue, hunger, and low blood sugar. The three Ps—polydipsia, polyphagia, and polyuria—are observed in hyperglycemia.

When a newly diagnosed type 2 diabetes mellitus patient asks the nurse why she has to take a pill instead of insulin, you reply that in type 2 diabetes, the body makes insulin but: a. overweight and underactive people simply cannot use the insulin produced. b. metabolism is slowed in some people so they have to take a pill to speed up their metabolism. c. sometimes the autoimmune system works against the action of the insulin. d. the cells become resistant to the action of insulin. Pills are given to increase the sensitivity.

ANS: D Type 2 diabetes mellitus is a disease in which the cells become resistant to the action of insulin and the blood glucose level rises. Oral hyperglycemic agents make the cells more sensitive.

The benefits of using an insulin pump include all of the following except: a. By continuously providing insulin they eliminate the need for injections of insulin b. They simplify management of blood sugar and often improve A1C c. They enable exercise without compensatory carbohydrate consumption d. They help with weight loss

ANS: D Using an insulin pump has many advantages, including fewer dramatic swings in blood glucose levels, increased flexibility about diet, and improved accuracy of insulin doses and delivery; however, the use of an insulin pump has been associated with weight gain.

The plan of care for a diabetic patient includes all of these interventions. Which intervention should you delegate to a UAP? 1. Checking to make sure that the patient's bath water is not too hot 2. Discussing community resources for diabetic outpatient care 3. Teaching the patient to perform daily foot inspection 4. Assessing the patient's technique for drawing insulin into a syringe

Ans: 1 Checking the bath water temperature is part of assisting with activities of daily living and is within the education and scope of practice of the UAP. Discussing community resources, teaching, and assessing require a higher level of education and are appropriate to the scope of practice of licensed nurses. Focus: Delegation; QSEN: TC; Concept: Safety

You are caring for a diabetic patient who is developing diabetic ketoacidosis (DKA). Which task delegation is most appropriate? 1. Ask the unit clerk to page the physician to come to the unit. 2. Ask the LPN/LVN to administer IV push insulin according to a sliding scale. 3. Ask the UAP to hang a new bag of normal saline. 4. Ask the UAP to get the patient a cup of orange juice.

Ans: 1 The nurse should not leave the patient. The scope of the unit clerk's job includes calling and paging physicians. LPNs/LVNs generally do not administer IV push medication. IV fluid administration is not within the scope of practice of UAPs. Patients with DKA already have a high glucose level and do not need orange juice. Focus: Delegation, supervision; QSEN: TC, S; Concept: Safety

An LPN/LVN is to administer rapid-acting insulin (Lispro) to a patient with type 1 diabetes. What essential information would you be sure to tell the LPN/LVN? 1. Give this insulin after the patient's food tray has been delivered and the patient is ready to eat. 2. Only give this insulin if the patient's fingerstick glucose reading is above 200 mg/dL. 3. This insulin mimics the basal glucose control of the pancreas. 4. Rapid-acting insulin is the only insulin that can be given subcutaneously or IV.

Ans: 1 The onset of action for rapid-acting insulin is within minutes, so it should be given only when the patient has food and is ready to eat. Because of this, rapid-acting insulin is sometimes called "see food" insulin. Options 2, 3, and 4 are incorrect. Long-acting insulins mimic the action of the pancreas. Regular insulin is the only insulin that can be given IV. Focus: Assignment, supervision; QSEN: TC, S; Concept: Communication

You are orienting a new graduate nurse who is providing diabetes education for a patient about insulin injection. For which teaching statement by the new nurse must you intervene? 1. "To prevent lipohypertrophy, be sure to rotate injection sites from the abdomen to the thighs." 2. "To correctly inject the insulin, lightly grasp a fold of skin and inject at a 90-degree angle." 3. "Always draw your regular insulin into the syringe first before your NPH insulin." 4. "Avoid injecting the insulin into scarred sites because those areas slow the absorption rate of insulin."

Ans: 1 While it is important to rotate injection sites for insulin, it is preferred that the injection sites be rotated within one anatomic site (e.g., the abdomen) to prevent day-to-day changes in the absorption rate of the insulin. All of the other teaching points are appropriate. Focus: Supervision, prioritization; QSEN: PCC, S; Concept: Leadership

A nursing diagnosis for a patient with newly-diagnosed diabetes is Risk for Injury related to sensory alterations. Which key points should you include in the teaching plan for this patient? (Select all that apply.) 1. "Clean and inspect your feet every day." 2. "Be sure that your shoes fit properly." 3. "Nylon socks are best to prevent friction on your toes from shoes." 4. "Only a podiatrist should trim your toenails." 5. "Report any nonhealing skin breaks to your health care provider."

Ans: 1, 2, 5 Sensory alterations are the major cause of foot complications in diabetic patients, and patients should be taught to examine their feet on a daily basis. Properly-fitted shoes protect the patient from foot complications. Broken skin increases the risk of infection. Cotton socks are recommended to absorb moisture. Patients, family, or health care providers may trim toenails. Focus: Prioritization; QSEN: PCC, S; Concept: Patient Education

You are caring for an 81-year-old adult with type 2 diabetes, hypertension, and peripheral vascular disease. Which admission assessment findings increase the patient's risk for development of hyperglycemic-hyperosmolar syndrome (HHS)? (Select all that apply.) 1. Hydrochlorothiazide (HCTZ) prescribed to control her diabetes 2. Weight gain of 6 pounds over the past month 3. Avoids consuming liquids in the evening 4. Blood pressure of 168/94 mm Hg 5. Urine output of 50 to 75 mL/hr

Ans: 1, 3 HHS often occurs in older adults with type 2 diabetes. Risk factors include taking diuretics and inadequate fluid intake. Weight loss (not weight gain) would be a symptom. While the patient's blood pressure is high, this is not a risk factor. A urine output of 50 to 75 mL/hr is adequate. Focus: Prioritization; QSEN: S; Concept: Clinical Judgment

In the care of a patient with type 2 diabetes, which actions can you delegate to a UAP? (Select all that apply.) 1. Providing the patient with extra packets of artificial sweetener for coffee 2. Assessing how well the patient's shoes fit 3. Recording the liquid intake from the patient's breakfast tray 4. Teaching the patient what to do if dizziness or lightheadedness occurs 5. Checking and recording the patient's blood pressure

Ans: 1, 3, 5 Giving the patient extra sweetener, recording oral intake, and checking blood pressure are all within the scope of practice of the UAP. Assessing shoe fit and patient teaching are not within the UAP's scope of practice. Focus: Assignment; QSEN: TC, S; Concept: Collaboration

A 30-yr-old patient has a new diagnosis of type 2 diabetes. The nurse will discuss the need to schedule a dilated eye examination a. every 2 years. c. when the patient is 39 years old. b. as soon as possible. d. within the first year after diagnosis.

B Because many patients have some diabetic retinopathy when they are first diagnosed with type 2 diabetes, a dilated eye examination is recommended at the time of diagnosis and annually thereafter. Patients with type 1 diabetes should have dilated eye examinations starting 5 years after they are diagnosed and then annually.

After change-of-shift report, which patient should the nurse assess first? a. A 19-yr-old patient with type 1 diabetes who has a hemoglobin A1C of 12% b. A 23-yr-old patient with type 1 diabetes who has a blood glucose of 40 mg/dL c. A 40-yr-old patient who is pregnant and whose oral glucose tolerance test is 202 mg/dL d. A 50-yr-old patient who uses exenatide (Byetta) and is complaining of acute abdominal pain

B Because the brain requires glucose to function, untreated hypoglycemia can cause unconsciousness, seizures, and death. The nurse will rapidly assess and treat the patient with low blood glucose. The other patients also have symptoms that require assessments or interventions, but they are not at immediate risk for life-threatening complications.

The nurse is preparing to teach a 43-yr-old man who is newly diagnosed with type 2 diabetes about home management of the disease. Which action should the nurse take first? a. Ask the patient's family to participate in the diabetes education program. b. Assess the patient's perception of what it means to have diabetes mellitus. c. Demonstrate how to check glucose using capillary blood glucose monitoring. d. Discuss the need for the patient to actively participate in diabetes management.

B Before planning teaching, the nurse should assess the patient's interest in and ability to self- manage the diabetes. After assessing the patient, the other nursing actions may be appropriate, but planning needs to be individualized to each patient.

Which patient action indicates good understanding of the nurse's teaching about administration of aspart (NovoLog) insulin? a. The patient avoids injecting the insulin into the upper abdominal area. b. The patient cleans the skin with soap and water before insulin administration. c. The patient stores the insulin in the freezer after administering the prescribed dose. d. The patient pushes the plunger down while removing the syringe from the injection site.

B Cleaning the skin with soap and water is acceptable. Insulin should not be frozen. The patient should leave the syringe in place for about 5 seconds after injection to be sure that all the insulin has been injected. The upper abdominal area is one of the preferred areas for insulin injection.

12.An occupational health nurse is screening a group of workers for diabetes. What statement should the nurse interpret as suggestive of diabetes? A) Ive always been a fan of sweet foods, but lately Im turned off by them. B) Lately, I drink and drink and cant seem to quench my thirst. C) No matter how much sleep I get, it seems to take me hours to wake up. D) When I went to the washroom the last few days, my urine smelled odd.

B Feedback: Classic clinical manifestations of diabetes include the three Ps: polyuria, polydipsia, and polyphagia. Lack of interest in sweet foods, fatigue, and foul-smelling urine are not suggestive of diabetes.

Which information will the nurse include when teaching a patient who has type 2 diabetes about glyburide ? a. Glyburide decreases glucagon secretion from the pancreas. b. Glyburide stimulates insulin production and release from the pancreas. c. Glyburide should be taken even if the morning blood glucose level is low. d. Glyburide should not be used for 48 hours after receiving IV contrast media.

B The sulfonylureas stimulate the production and release of insulin from the pancreas. If the glucose level is low, the patient should contact the health care provider before taking glyburide because hypoglycemia can occur with this class of medication. Metformin should be held for 48 hours after administration of IV contrast media, but this is not necessary for glyburide. Glucagon secretion is not affected by glyburide.

An active 32-yr-old male who has type 1 diabetes is being seen in the endocrine clinic. Which finding indicates a need for the nurse to discuss a possible a change in therapy with the health care provider? a. Hemoglobin A1C level of 6.2% b. Blood pressure of 140/88 mmHg c. Heart rate at rest of 58 beats/minute d. High density lipoprotein (HDL) level of 65 mg/dL

B To decrease the incidence of macrovascular and microvascular problems in patients with diabetes, the goal blood pressure is usually 130/80 mm Hg. An A1C less than 6.5%, a low resting heart rate (consistent with regular aerobic exercise in a young adult), and an HDL level of 65 mg/dL all indicate that the patient's diabetes and risk factors for vascular disease are well controlled.

A patient who has diabetes and reported burning foot pain at night receives a new prescription. Which information should the nurse teach the patient about amitriptyline ? a. Amitriptyline decreases the depression caused by your foot pain. b. Amitriptyline helps prevent transmission of pain impulses to the brain. c. Amitriptyline corrects some of the blood vessel changes that cause pain. d. Amitriptyline improves sleep and makes you less aware of nighttime pain.

B Tricyclic antidepressants (TCAs) decrease the transmission of pain impulses to the spinal cord and brain. TCAs also improve sleep quality and are used for depression, but that is not the major purpose for their use in diabetic neuropathy. The blood vessel changes that contribute to neuropathy are not affected by TCAs.

A patient receives aspart (NovoLog) insulin at 8:00 AM. At which time would the nurse anticipate the highest risk for hypoglycemia? a. 10:00 AM c. 2:00 PM b. 12:00 AM d. 4:0 PM

A The rapid-acting insulins peak in 1 to 3 hours. The patient is not at a high risk for hypoglycemia at the other listed times, although hypoglycemia may occur.

40.A patient is brought to the emergency department by the paramedics. The patient is a type 2 diabetic and is experiencing HHS. The nurse should identify what components of HHS? Select all that apply. A) Leukocytosis B) Glycosuria C) Dehydration D) Hypernatremia E) Hyperglycemia

B, C, D, E Feedback: In HHS, persistent hyperglycemia causes osmotic diuresis, which results in losses of water and electrolytes. To maintain osmotic equilibrium, water shifts from the intracellular fluid space to the extracellular fluid space. With glycosuria and dehydration, hypernatremia and increased osmolarity occur. Leukocytosis does not take place.

To monitor for complications in a patient with type 2 diabetes, which tests will the nurse in the diabetic clinic schedule at least annually (select all that apply)? a. Chest x-ray b. Blood pressure c. Serum creatinine d. Urine for microalbuminuria e. Complete blood count (CBC) f. Monofilament testing of the foot

B, C, D, F Blood pressure, serum creatinine, urine testing for microalbuminuria, and monofilament testing of the foot are recommended at least annually to screen for possible microvascular and macrovascular complications of diabetes. Chest x-ray and CBC might be ordered if the patient with diabetes presents with symptoms of respiratory or infectious problems but are not routinely included in screening.

After the nurse has finished teaching a patient who has a new prescription for exenatide (Byetta), which patient statement indicates that the teaching has been effective? a. "I may feel hungrier than usual when I take this medicine." b. "I will not need to worry about hypoglycemia with the Byetta." c. "I should take my daily aspirin at least an hour before the Byetta." d. "I will take the pill at the same time I eat breakfast in the morning."

C Because exenatide slows gastric emptying, oral medications should be taken at least 1 hour before the exenatide to avoid slowing absorption. Exenatide is injected and increases feelings of satiety. Hypoglycemia can occur with this medication.

3.A patient newly diagnosed with type 2 diabetes is attending a nutrition class. What general guideline would be important to teach the patients at this class? A) Low fat generally indicates low sugar. B) Protein should constitute 30% to 40% of caloric intake. C) Most calories should be derived from carbohydrates. D) Animal fats should be eliminated from the diet.

C Feedback: Currently, the ADA and the Academy of Nutrition and Dietetics (formerly the American Dietetic Association) recommend that for all levels of caloric intake, 50% to 60% of calories should be derived from carbohydrates, 20% to 30% from fat, and the remaining 10% to 20% from protein.Low fat does not automatically mean low sugar. Dietary animal fat does not need to be eliminated from the diet.

4.A nurse is providing health education to an adolescent newly diagnosed with type 1 diabetes mellitus and her family. The nurse teaches the patient and family that which of the following nonpharmacologic measures will decrease the bodys need for insulin? A) Adequate sleep B) Low stimulation C) Exercise D) Low-fat diet

C Feedback: Exercise lowers blood glucose, increases levels of HDLs, and decreases total cholesterol and triglyceride levels. Low fat intake and low levels of stimulation do not reduce a patients need for insulin. Adequate sleep is beneficial in reducing stress, but does not have an effect that is pronounced as that of exercise.

35.A patient with type 1 diabetes mellitus is seeing the nurse to review foot care. What would be a priority instruction for the nurse to give the patient? A) Examine feet weekly for redness, blisters, and abrasions. B) Avoid the use of moisturizing lotions. C) Avoid hot-water bottles and heating pads. D) Dry feet vigorously after each bath.

C Feedback: High-risk behaviors, such as walking barefoot, using heating pads on the feet, wearing open-toed shoes, soaking the feet, and shaving calluses, should be avoided. Socks should be worn for warmth. Feet should be examined each day for cuts, blisters, swelling, redness, tenderness, and abrasions. Lotion should be applied to dry feet but never between the toes. After a bath, the patient should gently, not vigorously, pat feet dry to avoid injury.

20.A 28-year-old pregnant woman is spilling sugar in her urine. The physician orders a glucose tolerance test, which reveals gestational diabetes. The patient is shocked by the diagnosis, stating that she is conscientious about her health, and asks the nurse what causes gestational diabetes. The nurse should explain that gestational diabetes is a result of what etiologic factor? A) Increased caloric intake during the first trimester B) Changes in osmolality and fluid balance C) The effects of hormonal changes during pregnancy D) Overconsumption of carbohydrates during the first two trimesters

C Feedback: Hyperglycemia and eventual gestational diabetes develops during pregnancy because of the secretion of placental hormones, which causes insulin resistance. The disease is not the result of food intake or changes in osmolality.

32.A student with diabetes tells the school nurse that he is feeling nervous and hungry. The nurse assesses the child and finds he has tachycardia and is diaphoretic with a blood glucose level of 50 mg/dL (2.8 mmol/L). What should the school nurse administer? A) A combination of protein and carbohydrates, such as a small cup of yogurt B) Two teaspoons of sugar dissolved in a cup of apple juice C) Half of a cup of juice, followed by cheese and crackers D) Half a sandwich with a protein-based filling

C Feedback: Initial treatment for hypoglycemia is 15 g concentrated carbohydrate, such as two or three glucose tablets, 1 tube glucose gel, or 0.5 cup juice. After initial treatment, the nurse should follow with a snack including starch and protein, such as cheese and crackers, milk and crackers, or half of a sandwich. It is unnecessary to add sugar to juice, even it if is labeled as unsweetened juice, because the fruit sugar in juice contains enough simple carbohydrate to raise the blood glucose level and additional sugar may result in a sharp rise in blood sugar that will last for several hours.

Which action by a patient indicates that the home health nurse's teaching about glargine and regular insulin has been successful? a. The patient administers the glargine 30 minutes before each meal. b. The patient's family prefills the syringes with the mix of insulins weekly. c. The patient discards the open vials of glargine and regular insulin after 4 weeks. d. The patient draws up the regular insulin and then the glargine in the same syringe.

C Insulin can be stored at room temperature for 4 weeks. Glargine should not be mixed with other insulins or prefilled and stored. Short-acting regular insulin is administered before meals, and glargine is given once daily.

When a patient with type 2 diabetes is admitted for a cholecystectomy, which nursing action can the nurse delegate to a licensed practical/vocational nurse (LPN/LVN)? a. Communicate the blood glucose level and insulin dose to the circulating nurse in surgery. b. Discuss the reason for the use of insulin therapy during the immediate postoperative period. c. Administer the prescribed lispro (Humalog) insulin before transporting the patient to surgery. d. Plan strategies to minimize the risk for hypoglycemia or hyperglycemia during the postoperative period.

C LPN/LVN education and scope of practice includes administration of insulin. Communication about patient status with other departments, planning, and patient teaching are skills that require RN education and scope of practice.

A few weeks after an 82-yr-old patient with a new diagnosis of type 2 diabetes has been placed on metformin (Glucophage) therapy and taught about appropriate diet and exercise, the home health nurse makes a visit. Which finding should the nurse promptly discuss with the health care provider? a. Hemoglobin A1C level is 7.9%. b. Last eye examination was 18 months ago. c. Glomerular filtration rate is decreased. d. Patient has questions about the prescribed diet.

C The decrease in renal function may indicate a need to adjust the dose of metformin or change to a different medication. In older patients, the goal for A1C may be higher in order to avoid complications associated with hypoglycemia. The nurse will plan on scheduling the patient for an eye examination and addressing the questions about diet, but the area for prompt intervention is the patient's decreased renal function.

A patient with type 2 diabetes is scheduled for a follow-up visit in the clinic several months from now. Which test will the nurse schedule to evaluate the effectiveness of treatment for the patient? a. Fasting blood glucose c. Glycosylated hemoglobin b. Oral glucose tolerance d. Urine dipstick for glucose

C The glycosylated hemoglobin (A1C) test shows the overall control of glucose over 90 to 120 days. A fasting blood level indicates only the glucose level at one time. Urine glucose testing is not an accurate reflection of blood glucose level and does not reflect the glucose over a prolonged time. Oral glucose tolerance testing is done to diagnose diabetes but is not used for monitoring glucose control after diabetes has been diagnosed.

A patient who was admitted with diabetic ketoacidosis secondary to a urinary tract infection has been weaned off an insulin drip 30 minutes ago. The patient reports feeling lightheaded and sweaty. Which action should the nurse take first? a. Infuse dextrose 50% by slow IV push. b. Administer 1 mg glucagon subcutaneously. c. Obtain a glucose reading using a finger stick. d. Have the patient drink 4 ounces of orange juice.

C The patient's clinical manifestations are consistent with hypoglycemia, and the initial action should be to check the patient's glucose with a finger stick or order a stat blood glucose. If the glucose is low, the patient should ingest a rapid-acting carbohydrate, such as orange juice. Glucagon or dextrose 50% might be given if the patient's symptoms become worse or if the patient is unconscious.

After change-of-shift report, which patient will the nurse assess first? a. A 19-yr-old patient with type 1 diabetes who was admitted with possible daw phenomenon b. A 35-yr-old patient with type 1 diabetes whose most recent blood glucose reading was 230 mg/dL c. A 60-yr-old patient with hyperosmolar hyperglycemic syndrome who has poor skin turgor and dry oral mucosa d. A 68-yr-old patient with type 2 diabetes who has severe peripheral neuropathy and complains of burning foot pain

C The patient's diagnosis of HHS and signs of dehydration indicate that the nurse should rapidly assess for signs of shock and determine whether increased fluid infusion is needed. The other patients also need assessment and intervention but do not have life-threatening complications.

A patient screened for diabetes at a clinic has a fasting plasma glucose level of 120 mg/dL (6.7 mmol/L). The nurse will plan to teach the patient about a. self-monitoring of blood glucose. b. using low doses of regular insulin. c. lifestyle changes to lower blood glucose. d. effects of oral hypoglycemic medications.

C The patient's impaired fasting glucose indicates prediabetes, and the patient should be counseled about lifestyle changes to prevent the development of type 2 diabetes. The patient with prediabetes does not require insulin or oral hypoglycemics for glucose control and does not need to self- monitor blood glucose.

A patient who has type 2 diabetes is being prepared for an elective coronary angiogram. Which information would the nurse anticipate might lead to rescheduling the test? a. The patient's most recent A1C was 6.5%. b. The patient's blood glucose is 128 mg/dL. c. The patient took the prescribed metformin today. d. The patient took the prescribed captopril this morning.

C To avoid lactic acidosis, metformin should be discontinued a day or 2 before the coronary angiogram and should not be used for 48 hours after IV contrast media are administered. The other patient data will also be reported but do not indicate any need to reschedule the procedure.

The nurse is assessing a 22-yr-old patient experiencing the onset of symptoms of type 1 diabetes. To which question would the nurse anticipate a positive response? a. "Are you anorexic?" b. "Is your urine dark colored?" c. "Have you lost weight lately?" d. "Do you crave sugary drinks?"

C Weight loss occurs because the body is no longer able to absorb glucose and starts to break down protein and fat for energy. The patient is thirsty but does not necessarily crave sugar-containing fluids. Increased appetite is a classic symptom of type 1 diabetes. With the classic symptom of polyuria, urine will be very dilute.

25.A patient with type 2 diabetes has been managing his blood glucose levels using diet and metformin (Glucophage). Following an ordered increase in the patients daily dose of metformin, the nurse should prioritize which of the following assessments? A) Monitoring the patients neutrophil levels B) Assessing the patient for signs of impaired liver function C) Monitoring the patients level of consciousness and behavior D) Reviewing the patients creatinine and BUN levels

D Feedback: Metformin has the potential to be nephrotoxic; consequently, the nurse should monitor the patients renal function. This drug does not typically affect patients neutrophils, liver function, or cognition.

9.A school nurse is teaching a group of high school students about risk factors for diabetes. Which of the following actions has the greatest potential to reduce an individuals risk for developing diabetes? A) Have blood glucose levels checked annually. B) Stop using tobacco in any form. C) Undergo eye examinations regularly. D) Lose weight, if obese.

D Feedback: Obesity is a major modifiable risk factor for diabetes. Smoking is not a direct risk factor for the disease. Eye examinations are necessary for persons who have been diagnosed with diabetes, but they do not screen for the disease or prevent it. Similarly, blood glucose checks do not prevent the diabetes.

Which finding indicates a need to contact the health care provider before the nurse administers metformin (Glucophage)? a. The patient's blood glucose level is 174 mg/dL. b. The patient is scheduled for a chest x-ray in an hour. c. The patient has gained 2 lb (0.9 kg) in the past 24 hours. d. The patient's blood urea nitrogen (BUN) level is 52 mg/dL.

D The BUN indicates possible renal failure, and metformin should not be used in patients with renal failure. The other findings are not contraindications to the use of metformin.

The nurse has been teaching a patient with type 2 diabetes about managing blood glucose levels and taking glipizide (Glucotrol). Which patient statement indicates a need for additional teaching? a. "If I overeat at a meal, I will still take the usual dose of medication." b. "Other medications besides the Glucotrol may affect my blood sugar." c. "When I am ill, I may have to take insulin to control my blood sugar." d. "My diabetes won't cause complications because I don't need insulin."

D The patient should understand that type 2 diabetes places the patient at risk for many complications and that good glucose control is as important when taking oral agents as when using insulin. The other statements are accurate and indicate good understanding of the use of glipizide.

A patient with diabetic ketoacidosis is brought to the emergency department. Which prescribed action should the nurse implement first? a. Infuse 1 L of normal saline per hour. b. Give sodium bicarbonate 50 mEq IV push. c. Administer regular insulin 10 U by IV push. d. Start a regular insulin infusion at 0.1 units/kg/hr.

A The most urgent patient problem is the hypovolemia associated with diabetic ketoacidosis (DKA), and the priority is to infuse IV fluids. The other actions can be done after the infusion of normal saline is initiated.

Which information will the nurse include in teaching a female patient who has peripheral arterial disease, type 2 diabetes, and sensory neuropathy of the feet and legs? a. Choose flat-soled leather shoes. b. Set heating pads on a low temperature. c. Use callus remover for corns or calluses. d. Soak feet in warm water for an hour each day.

A The patient is taught to avoid high heels and that leather shoes are preferred. The feet should be washed, but not soaked, in warm water daily. Heating pad use should be avoided. Commercial callus and corn removers should be avoided. The patient should see a specialist to treat these problems.

A 58-year-old with type 2 diabetes was admitted to your unit with a diagnosis of chronic obstructive pulmonary disease (COPD) exacerbation. When you prepare a care plan for this patient, what would you be sure to include? (Select all that apply.) 1. Fingerstick blood glucose checks before meals and at bedtime 2. Sliding-scale insulin dosing as ordered 3. Bed rest until the COPD exacerbation is resolved 4. Teaching about the Atkins diet for weight loss 5. Demonstration of the components of foot care

6. Ans: 1, 2, 5 When a diabetic patient is ill, glucose levels become elevated, and administration of insulin may be necessary. Teaching or reviewing the components of proper foot care is always a good idea with a diabetic patient. Bed rest is not necessary, and glucose level may be better controlled when a patient is more active. The Atkins diet recommends decreasing the consumption of carbohydrates and is not a good diet for diabetic patients. Focus: Prioritization; QSEN: PCC, S; Concept: Glucose Regulation

Which question during the assessment of a patient who has diabetes will help the nurse identify autonomic neuropathy? a. "Do you feel bloated after eating?" b. "Have you seen any skin changes?" c. "Do you need to increase your insulin dosage when you are stressed?" d. "Have you noticed any painful new ulcerations or sores on your feet?"

A Autonomic neuropathy can cause delayed gastric emptying, which results in a bloated feeling for the patient. The other questions are also appropriate to ask but would not help in identifying autonomic neuropathy.

To assist an older patient with diabetes to engage in moderate daily exercise, which action is most important for the nurse to take? a. Determine what types of activities the patient enjoys. b. Remind the patient that exercise improves self-esteem. c. Teach the patient about the effects of exercise on glucose level. d. Give the patient a list of activities that are moderate in intensity.

A Because consistency with exercise is important, assessment for the types of exercise that the patient finds enjoyable is the most important action by the nurse in ensuring adherence to an exercise program. The other actions may be helpful but are not the most important in improving compliance.

Which nursing action can the nurse delegate to experienced unlicensed assistive personnel (UAP) who are working in the diabetic clinic? a. Measure the ankle-brachial index. b. Check for changes in skin pigmentation. c. Assess for unilateral or bilateral foot drop. d. Ask the patient about symptoms of depression.

A Checking systolic pressure at the ankle and brachial areas and calculating the ankle-brachial index is a procedure that can be done by UAP who have been trained in the procedure. The other assessments require more education and critical thinking and should be done by the registered nurse (RN).

10.A 15-year-old child is brought to the emergency department with symptoms of hyperglycemia and is subsequently diagnosed with diabetes. Based on the fact that the childs pancreatic beta cells are being destroyed, the patient would be diagnosed with what type of diabetes? A) Type 1 diabetes B) Type 2 diabetes C) Noninsulin-dependent diabetes D) Prediabetes

A Feedback: Beta cell destruction is the hallmark of type 1 diabetes. Noninsulin-dependent diabetes is synonymous with type 2 diabetes, which involves insulin resistance and impaired insulin secretion, but not beta cell destruction. Prediabetes is characterized by normal glucose metabolism, but a previous history of hyperglycemia, often during illness or pregnancy.

37.The most recent blood work of a patient with a longstanding diagnosis of type 1 diabetes has shown the presence of microalbuminuria. What is the nurses most appropriate action? A) Teach the patient about actions to slow the progression of nephropathy. B) Ensure that the patient receives a comprehensive assessment of liver function. C) Determine whether the patient has been using expired insulin. D) Administer a fluid challenge and have the test repeated.

A Feedback: Clinical nephropathy eventually develops in more than 85% of people with microalbuminuria. As such, educational interventions addressing this microvascular complication are warranted. Expired insulin does not cause nephropathy, and the patients liver function is not likely affected. There is no indication for the use of a fluid challenge.

2.A patient presents to the clinic complaining of symptoms that suggest diabetes. What criteria would support checking blood levels for the diagnosis of diabetes? A) Fasting plasma glucose greater than or equal to 126 mg/dL B) Random plasma glucose greater than 150 mg/dL C) Fasting plasma glucose greater than 116 mg/dL on 2 separate occasions D) Random plasma glucose greater than 126 mg/dL

A Feedback: Criteria for the diagnosis of diabetes include symptoms of diabetes plus random plasma glucose greater than or equal to 200 mg/dL, or a fasting plasma glucose greater than or equal to 126 mg/dL.

26.A patient with a longstanding diagnosis of type 1 diabetes has a history of poor glycemic control. The nurse recognizes the need to assess the patient for signs and symptoms of peripheral neuropathy. Peripheral neuropathy constitutes a risk for what nursing diagnosis? A) Infection B) Acute pain C) Acute confusion D) Impaired urinary elimination

A Feedback: Decreased sensations of pain and temperature place patients with neuropathy at increased risk for injury and undetected foot infections. The neurologic changes associated with peripheral neuropathy do not normally result in pain, confusion, or impairments in urinary function.

30.A nurse is conducting a class on how to self-manage insulin regimens. A patient asks how long a vial of insulin can be stored at room temperature before it goes bad. What would be the nurses best answer? A) If you are going to use up the vial within 1 month it can be kept at room temperature. B) If a vial of insulin will be used up within 21 days, it may be kept at room temperature. C) If a vial of insulin will be used up within 2 weeks, it may be kept at room temperature. D) If a vial of insulin will be used up within 1 week, it may be kept at room temperature.

A Feedback: If a vial of insulin will be used up within 1 month, it may be kept at room temperature.

27.A patient has been brought to the emergency department by paramedics after being found unconscious. The patients Medic Alert bracelet indicates that the patient has type 1 diabetes and the patients blood glucose is 22 mg/dL (1.2 mmol/L). The nurse should anticipate what intervention? A) IV administration of 50% dextrose in water B) Subcutaneous administration of 10 units of Humalog C) Subcutaneous administration of 12 to 15 units of regular insulin D) IV bolus of 5% dextrose in 0.45% NaCl

A Feedback: In hospitals and emergency departments, for patients who are unconscious or cannot swallow, 25 to 50 mL of 50% dextrose in water (D50W) may be administered IV for the treatment of hypoglycemia. Five percent dextrose would be inadequate and insulin would exacerbate the patients condition.

34.A patient has been living with type 2 diabetes for several years, and the nurse realizes that the patient is likely to have minimal contact with the health care system. In order to ensure that the patient maintains adequate blood sugar control over the long term, the nurse should recommend which of the following? A) Participation in a support group for persons with diabetes B) Regular consultation of websites that address diabetes management C) Weekly telephone check-ins with an endocrinologist D) Participation in clinical trials relating to antihyperglycemics

A Feedback: Participation in support groups is encouraged for patients who have had diabetes for many years as well as for those who are newly diagnosed. This is more interactive and instructive than simply consulting websites. Weekly telephone contact with an endocrinologist is not realistic in most cases. Participation in research trials may or may not be beneficial and appropriate, depending on patients circumstances.

28.A diabetic nurse is working for the summer at a camp for adolescents with diabetes. When providing information on the prevention and management of hypoglycemia, what action should the nurse promote? A) Always carry a form of fast-acting sugar. B) Perform exercise prior to eating whenever possible. C) Eat a meal or snack every 8 hours. D) Check blood sugar at least every 24 hours.

A Feedback: The following teaching points should be included in information provided to the patient on how to prevent hypoglycemia: Always carry a form of fast-acting sugar, increase food prior to exercise, eat a meal or snack every 4 to 5 hours, and check blood sugar regularly.

7.A diabetes nurse educator is teaching a group of patients with type 1 diabetes about sick day rules. What guideline applies to periods of illness in a diabetic patient? A) Do not eliminate insulin when nauseated and vomiting. B) Report elevated glucose levels greater than 150 mg/dL. C) Eat three substantial meals a day, if possible. D) Reduce food intake and insulin doses in times of illness.

A Feedback: The most important issue to teach patients with diabetes who become ill is not to eliminate insulin doses when nausea and vomiting occur. Rather, they should take their usual insulin or oral hypoglycemic agent dose, then attempt to consume frequent, small portions of carbohydrates. In general, blood sugar levels will rise but should be reported if they are greater than 300 mg/dL.

A female patient is scheduled for an oral glucose tolerance test. Which information from the patient's health history is important for the nurse to communicate to the health care provider regarding this test? a. The patient uses oral contraceptives. b. The patient runs several days a week. c. The patient has been pregnant three times. d. The patient has a family history of diabetes.

A Oral contraceptive use may falsely elevate oral glucose tolerance test (OGTT) values. Exercise and a family history of diabetes both can affect blood glucose but will not lead to misleading information from the OGTT. History of previous pregnancies may provide informational about gestational glucose tolerance but will not lead to misleading information from the OGTT.

Which statement by the patient indicates a need for additional instruction in administering insulin? a. "I need to rotate injection sites among my arms, legs, and abdomen each day." b. "I can buy the 0.5-mL syringes because the line markings will be easier to see." c. "I do not need to aspirate the plunger to check for blood before injecting insulin." d. "I should draw up the regular insulin first, after injecting air into the NPH bottle."

A Rotating sites is no longer recommended because there is more consistent insulin absorption when the same site is used consistently. The other patient statements are accurate and indicate that no additional instruction is needed.

The nurse has administered 4 oz of orange juice to an alert patient whose blood glucose was 62 mg/dL. Fifteen minutes later, the blood glucose is 67 mg/dL. Which action should the nurse take next? a. Give the patient 4 to 6 oz more orange juice. b. Administer the PRN glucagon (Glucagon) 1 mg IM. c. Have the patient eat some peanut butter with crackers. d. Notify the health care provider about the hypoglycemia.

A The "rule of 15" indicates that administration of quickly acting carbohydrates should be done two or three times for a conscious patient whose glucose remains less than 70 mg/dL before notifying the health care provider. More complex carbohydrates and fats may be used after the glucose has stabilized. Glucagon should be used if the patient's level of consciousness decreases so that oral carbohydrates can no longer be given.

A patient who has type 1 diabetes plans to swim laps for an hour daily at 1:00 PM. The clinic nurse will plan to teach the patient to a. check glucose level before, during, and after swimming. b. delay eating the noon meal until after the swimming class. c. increase the morning dose of neutral protamine Hagedorn (NPH) insulin. d. time the morning insulin injection so that the peak occurs while swimming.

A The change in exercise will affect blood glucose, and the patient will need to monitor glucose carefully to determine the need for changes in diet and insulin administration. Because exercise tends to decrease blood glucose, patients are advised to eat before exercising. Increasing the morning NPH or timing the insulin to peak during exercise may lead to hypoglycemia, especially with the increased exercise.

The nurse is assessing a 55-yr-old female patient with type 2 diabetes who has a body mass index (BMI) of 31 kg/m2 .Which goal in the plan of care is most important for this patient? a. The patient will reach a glycosylated hemoglobin level of less than 7%. b. The patient will follow a diet and exercise plan that results in weight loss. c. The patient will choose a diet that distributes calories throughout the day. d. The patient will state the reasons for eliminating simple sugars in the diet.

A The complications of diabetes are related to elevated blood glucose and the most important patient outcome is the reduction of glucose to near-normal levels. A BMI of 30?9?kg/m2 or above is considered obese, so the other outcomes are appropriate but are not as high in priority.

In which order will the nurse take these steps to prepare NPH 20 units and regular insulin 2 units using the same syringe? (Put a comma and a space between each answer choice [A, B, C, D, E]). a. Rotate NPH vial. b. Withdraw regular insulin. c. Withdraw 20 units of NPH. d. Inject 20 units of air into NPH vial. e. Inject 2 units of air into regular insulin vial.

ANS: A, D, E, B, C When mixing regular insulin with NPH, it is important to avoid contact between the regular insulin and the additives in the NPH that slow the onset, peak, and duration of activity in the longer-acting insulin.

The nurse is administering metformin (Glucophage) to a client. What nursing observations would cause the nurse concern regarding side effects of the medication? a. Gastrointestinal upset b. Photophobia c. Hyperglycemia d. Skin eruptions

ANS: A Anorexia, nausea, and a metallic taste in the mouth are common side effects, but can contribute to the client not taking the medication if unaware of the expected side effects. Over time, the gastrointestinal symptoms subside and can be relieved by taking the medication with food or by starting at a lower dose.

A client who has been taking pioglitazone (Actos) for 6 months reports to the nurse that his urine has become darker since starting the medication. Which is the nurse's first action? a. Review results of liver enzyme studies. b. Document the report in the client's chart. c. Instruct the client to increase water intake. d. Test a sample of urine for occult blood.

ANS: A Thiazolidinediones (including pioglitazone) can affect liver function; liver function should be assessed at the start of therapy and at regular intervals while the client continues to take these drugs. Dark urine is one indicator of liver impairment because bilirubin is increased in the blood and is excreted in the urine. The nurse should check the client's most recent liver function studies. Documentation should be done after all assessments have been completed. The client does not need to be told to increase water intake, and the nurse does not need to check the urine for occult blood.

Glipizide (Glucotrol) 10 mg bid PO has been ordered for an adult client with type 2 diabetes. The nurse would explain to the client that the medication reduces the blood sugar level by what process? a. Delays the cellular uptake of potassium and insulin b. Stimulates insulin release from the pancreas c. Decreases the body's need for and utilization of insulin at the cellular level d. Interferes with the absorption and metabolism of fats and carbohydrates

ANS: B The sulfonylureas reduce the blood glucose level by stimulating insulin release from the pancreas. Over a long period of time, sulfonylureas may actually increase insulin effects at the cellular level and decrease glucose production by the liver. This is the reason that sulfonylureas are prescribed for clients with type 2 diabetes who still have a functioning pancreas.

The labor nurse is providing care to a patient at 37 weeks' gestation who is an insulin-dependent diabetic. The health care provider prescribes an infusion of insulin throughout her induction to be titrated to keep her blood glucose levels below 110 mg/dL. What type of insulin will the nurse select to prepare the infusion? a. NPH insulin b. Regular insulin c. Lispro (Humalog) d. Aspart (Novolog)

ANS: B Continuous infusion of a regular insulin solution combined with a separate intravenous solution containing glucose, such as 5% dextrose in Ringer's lactate, allows titration to maintain blood glucose levels between 80 and 110 mg/dL, or as designated by facility policy. The insulin solution is raised, lowered, or discontinued to maintain euglycemia based on hourly capillary blood glucose levels.

The nurse has given a client an injection of glucagon. Which action does the nurse take next? a. Apply pressure to the injection site. b. Position the client on his or her side. c. Have a padded tongue blade available. d. Elevate the head of the bed.

ANS: B Glucagon administration often induces vomiting, increasing the client's risk for aspiration. The other actions are not required.

If dietary trays are usually brought to the nursing unit at 8:00 am, the nurse should plan to administer intermediate-acting insulin (Humulin N), 40 units, subcutaneously to a client between: a. 5:00 and 5:30 am b. 6:30 and 7:00 am c. 9:30 and 10:30 am d. 11:00 and 11:30 am

ANS: B Intermediate-acting insulin, such as Humulin N, should be given 60 to 90 minutes before a meal. Therefore, if the breakfast tray arrived at 8:00 am, a client would need to receive the insulin between 6:30 and 7:30 am. Regular insulin usually is administered 30 minutes before a meal, and insulin lispro is given immediately (15 minutes) before or after meals.

The nurse administers 6 units of regular insulin and 10 units NPH insulin at 7 AM. At what time does the nurse assess the client for problems related to the NPH insulin? a. 8 AM b. 4 PM c. 8 PM d. 11 PM

ANS: B NPH is an intermediate-acting insulin with an onset of 1.5 hours, peak of 4 to 12 hours, and duration of action of 22 hours. Checking the client at 8:00 AM would be too soon; 8:00 PM and 11:00 PM would be too late.

Which statement made by a client with type 2 diabetes taking nateglinide (Starlix) indicates understanding of this therapy? a. "I'll take this medicine with my meals." b. "I'll take this medicine right before I eat." c. "I'll take this medicine just before I go to bed." d. "I'll take this medicine when I wake up in the morning."

ANS: B Nateglinide is a meglitinide that is designed to increase meal-related insulin secretion. It should be taken just before a meal. The other options are incorrect.

A client who has type 2 diabetes is prescribed glipizide (Glucotrol). Which precautions does the nurse include in the teaching plan related to this medication? a. "Change positions slowly when you get up." b. "Avoid taking nonsteroidal anti-inflammatory drugs." c. "If you miss a dose of this drug, you can double the next dose." d. "Discontinue the medication if you develop an infection."

ANS: B Nonsteroidal anti-inflammatory drugs potentiate the hypoglycemic effects of sulfonylurea agents. Glipizide is a sulfonylurea. The other statements are not applicable to glipizide.

The nurse understands the following about the correct administration of insulin lispro: a. It needs to be taken after the meals. b. It should be taken within 15 minutes of beginning a meal. c. It is to be taken once daily at the noon meal. d. It is taken only in the evenings with a snack before bedtime.

ANS: B Rapid-acting insulins, such as insulin lispro (Humalog) and insulin aspart (Novolog), are able to more closely mimic the body's natural rapid insulin output after consumption of a meal, which is why both medications usually are administered within 15 minutes of beginning a meal.

A client with diabetes receives a combination of regular and NPH insulin at 0700 hours. The nurse teaches the client to be alert for signs of hypoglycemia at: a. 12 pm and 1 pm (1200 and 1300 hours) b. 9am and 5pm (0900 and 1700 hours) c. 10 am and 10 pm (1000 and 2200 hours) d. 8am and 11 am (0800 and 1100 hours)

ANS: B Regular insulin (a short-acting insulin) peaks in 2 to 3 hours, and NPH (an intermediate-acting insulin) peaks in 4 to 10 hours. Hypoglycemia would most likely occur between 9 am and 5 pm (0900 to 1700 hours).

The nurse has been reviewing options for insulin therapy with several clients. For which client does the nurse choose to recommend the pen-type injector insulin delivery system? a. Older adult client who lives at home alone but has periods of confusion b. Client on an intensive regimen with frequent, small insulin doses c. Client from the low-vision clinic who has trouble seeing the syringe d. "Brittle" client who has frequent episodes of hypoglycemia

ANS: B The pen-type injector allows greater accuracy with small doses, especially doses lower than 5 units. It is not recommended for those who have visual or neurologic impairments. The client with frequent hypoglycemia would not derive special benefit from using the pen.

The nurse identifies a need for additional teaching when the patient who is self- monitoring blood glucose a. washes the puncture site using warm water and soap. b. chooses a puncture site in the center of the finger pad. c. hangs the arm down for a minute before puncturing the site. d. says the result of 120 mg indicates good blood sugar control.

B The patient is taught to choose a puncture site at the side of the finger pad because there are fewer nerve endings along the side of the finger pad. The other patient actions indicate that teaching has been effective.

A patient has just been diagnosed with diabetes mellitus. His doctor has requested glucagon for emergency use at home. The nurse instructs the patient that the purpose of this drug is to treat: a. Hyperglycemia from insufficient insulin injection. b. Hyperglycemia from eating a large meal. c. Hypoglycemia from insulin overdose. d. Lipohypertrophy from inadequate insulin absorption.

ANS: C Glucagon is for emergency use for insulin overdose. The patient will usually arouse within 20 minutes if unconscious. The family should also be instructed how to use the glucagon injection as well.

Which of the following diabetes drugs acts by decreasing the amount of glucose produced by the liver? a. Sulfonylureas b. Meglitinides c. Biguanides d. Alpha-glucosidase inhibitors

ANS: C Biguanides, such as metformin, lower blood glucose by reducing the amount of glucose produced by the liver. Sulfonylureas and Meglitinides stimulate the beta cells of the pancreas to produce more insulin. Alpha-glucosidase inhibitors block the breakdown of starches and some sugars, which helps to reduce blood glucose levels

It is important for the nurse to teach the client which of the following about metformin (Glucophage)? a. It may cause constipation b. It should be taken at night c. It should be taken with meals d. It may increase the effects of aspirin

ANS: C Metformin (Glucophage) is administered with meals to minimize gastrointestinal effects. These adverse effects are abdominal bloating, diarrhea, nausea, vomiting, and an unpleasant metallic taste. Metformin interacts with alcohol and cimetidine and is contraindicated in heart failure and liver disease and in clients with compromised renal function

You are preparing to review a teaching plan for a patient with type 2 diabetes mellitus. To determine the patient's level of compliance with his prescribed diabetic regimen, which value would you be sure to review? 1. Fasting glucose level 2. Oral glucose tolerance test results 3. Glycosylated hemoglobin (HgbA1c) level 4. Fingerstick glucose findings for 24 hours

. Ans: 3 The higher the blood glucose level is over time, the more glycosylated the hemoglobin becomes. The HgbA1c level is a good indicator of the average blood glucose level over the previous 120 days. Fasting glucose and oral glucose tolerance tests are important diagnostic tools. Fingerstick blood glucose monitoring provides information that allows adjustment of the patient's therapeutic regimen. Focus: Prioritization; QSEN: EBP; Concept: Adherence

The patient with type 2 diabetes is "nothing by mouth" (NPO) for a cardiac catheterization. An LPN/LVN who is administering medications to this patient asks you (the supervising RN) whether the patient should receive his ordered repaglinide (Prandin). What is your best response? 1. "Yes, because this drug will increase the patient's insulin secretion and prevent hyperglycemia." 2. "No, because this drug may cause the patient to experience gastrointestinal symptoms such as nausea." 3. "No, because this drug should be given 1 to 30 minutes before meals and the patient is NPO." 4. "Yes, because this drug should be taken 3 times a day whether the patient eats or not."

24. Ans: 3 Repaglinide is a meglitinide analog drug. These drugs are short-acting agents used to prevent postmeal blood glucose elevation. They should be given within 1 to 30 minutes before meals and cause hypoglycemia shortly after dosing when a meal is delayed or omitted. Focus: Supervision, delegation, prioritization; QSEN: PCC, TC, S; Concept: Communication

18.A diabetic educator is discussing sick day rules with a newly diagnosed type 1 diabetic. The educator is aware that the patient will require further teaching when the patient states what? A) I will not take my insulin on the days when I am sick, but I will certainly check my blood sugar every 2 hours. B) If I cannot eat a meal, I will eat a soft food such as soup, gelatin, or pudding six to eight times a day. C) I will call the doctor if I am not able to keep liquids in my body due to vomiting or diarrhea. D) I will call the doctor if my blood sugar is over 300 mg/dL or if I have ketones in my urine.

A Feedback: The nurse must explanation the sick day rules again to the patient who plans to stop taking insulin when sick. The nurse should emphasize that the patient should take insulin agents as usual and test ones blood sugar and urine ketones every 3 to 4 hours. In fact, insulin-requiring patients may need supplemental doses of regular insulin every 3 to 4 hours. The patient should report elevated glucose levels (greater than 300 mg/dL or as otherwise instructed) or urine ketones to the physician. If the patient is not able to eat normally, the patient should be instructed to substitute soft foods such a gelatin, soup, and pudding. If vomiting, diarrhea, or fever persists, the patient should have an intake of liquids every 30 to 60 minutes to prevent dehydration.

22.A nurse is caring for a patient newly diagnosed with type 1 diabetes. The nurse is educating the patient about self-administration of insulin in the home setting. The nurse should teach the patient to do which of the following? A) Avoid using the same injection site more than once in 2 to 3 weeks. B) Avoid mixing more than one type of insulin in a syringe. C) Cleanse the injection site thoroughly with alcohol prior to injecting. D) Inject at a 45 angle.

A Feedback: To prevent lipodystrophy, the patient should try not to use the same site more than once in 2 to 3 weeks. Mixing different types of insulin in a syringe is acceptable, within specific guidelines, and the needle is usually inserted at a 90 angle. Cleansing the injection site with alcohol is optional.

A 27-yr-old patient admitted with diabetic ketoacidosis (DKA) has a serum glucose level of 732 mg/dL and serum potassium level of 3.1 mEq/L. Which action prescribed by the health care provider should the nurse take first? a. Place the patient on a cardiac monitor. b. Administer IV potassium supplements. c. Ask the patient about home insulin doses. d. Start an insulin infusion at 0.1 units/kg/hr.

A Hypokalemia can lead to potentially fatal dysrhythmias such as ventricular tachycardia and ventricular fibrillation, which would be detected with electrocardiogram (ECG) monitoring. Because potassium must be infused over at least 1 hour, the nurse should initiate cardiac monitoring before infusion of potassium. Insulin should not be administered without cardiac monitoring because insulin infusion will further decrease potassium levels. Discussion of home insulin and possible causes can wait until the patient is stabilized.

Which patient action indicates a good understanding of the nurse's teaching about the use of an insulin pump? a. The patient programs the pump for an insulin bolus after eating. b. The patient changes the location of the insertion site every week. c. The patient takes the pump off at bedtime and starts it again each morning. d. The patient plans a diet with more calories than usual when using the pump.

A In addition to the basal rate of insulin infusion, the patient will adjust the pump to administer a bolus after each meal, with the dosage depending on the oral intake. The insertion site should be changed every 2 or 3 days. There is more flexibility in diet and exercise when an insulin pump is used, but it does not provide for consuming a higher calorie diet. The pump will deliver a basal insulin rate 24 hours a day.

A patient with diabetes is starting on intensive insulin therapy. Which type of insulin will the nurse discuss using for mealtime coverage? a. Lispro (Humalog) c. Detemir (Levemir) b. Glargine (Lantus) d. NPH (Humulin N)

A Rapid- or short-acting insulin is used for mealtime coverage for patients receiving intensive insulin therapy. NPH, glargine, or detemir will be used as the basal insulin.

A client with diabetes is prescribed insulin glargine once daily and regular insulin four times daily. One dose of regular insulin is scheduled at the same time as the glargine. How does the nurse instruct the client to administer the two doses of insulin? a. "Draw up and inject the insulin glargine first, then draw up and inject the regular insulin." b. "Draw up and inject the insulin glargine first, wait 20 minutes, then draw up and inject the regular insulin." c. "First draw up the dose of regular insulin, then draw up the dose of insulin glargine in the same syringe, mix, and inject the two insulins together." d. "First draw up the dose of insulin glargine, then draw up the dose of regular insulin in the same syringe, mix, and inject the two insulins together."

ANS: A Insulin glargine must not be diluted or mixed with any other insulin or solution. Mixing results in an unpredictable alteration in the onset of action and time to peak action. The correct instruction is to draw up and inject first the glargine, then the regular insulin right afterward.

A client with diabetes is visually impaired and wants to know whether syringes can be prefilled and stored for later use. Which is the nurse's best response? a. "Yes. Prefilled syringes can be stored for 3 weeks in the refrigerator in a vertical position with the needle pointing up." b. "Yes. Prefilled syringes can be stored for up to 3 weeks in the refrigerator, placed in a horizontal position." c. "Insulin reacts with plastic, so prefilled syringes are okay, but they must be made of glass." d. "No. Insulin cannot be stored for any length of time outside of the container."

ANS: A Insulin is relatively stable when stored in a cool, dry place away from light. When refrigerated, prefilled syringes are stable for up to 3 weeks. They should be stored in the vertical position with the needle pointing up to prevent suspended insulin particles from clogging the needle. The other answers are inaccurate.

A nurse reviews the medication list of a client recovering from a computed tomography (CT) scan with IV contrast to rule out small bowel obstruction. Which medication should alert the nurse to contact the provider and withhold the prescribed dose? a. Pioglitazone (Actos) b. Glimepiride (Amaryl) c. Glipizide (Glucotrol) d. Metformin (Glucophage)

ANS: D Glucophage should not be administered when the kidneys are attempting to excrete IV contrast from the body. This combination would place the client at high risk for kidney failure. The nurse should hold the metformin dose and contact the provider. The other medications are safe to administer after receiving IV contrast.

A client has a new insulin pump. Which is the nurse's priority instruction in teaching the client? a. "Test your urine daily for ketones." b. "Use only buffered insulin." c. "Keep the insulin frozen until you need it." d. "Change the needle every 3 days."

ANS: D Having the same needle remain in place through the skin for longer than 3 days drastically increases the risk for infection in or through the delivery system. Having an insulin pump does not require the client to test for ketones in the urine. Insulin should not be frozen. Insulin is not buffered.

What would be important for the nurse to include in the teaching plan for clients who are taking insulin? a. The client should use only the injection sites that are most accessible. b. During times of illness, clients should increase their insulin dosage by 25%. c. When mixing insulins, the NPH insulin should be drawn up into the syringe first. d. When mixing insulins, regular insulin should be drawn up into the syringe first.

ANS: D If mixing insulins, the regular insulin should always be drawn up into the syringe first. Remember: clear to cloudy; regular insulin first, followed by cloudy ones, such as NPH and Ultralente. Clients should always rotate injection sites (preferably in the abdomen) and should notify their physicians if they become ill.

A client is receiving IV insulin for hyperglycemia. Which laboratory value requires immediate intervention by the nurse? a. Serum chloride level of 98 mmol/L b. Serum calcium level of 8.8 mg/dL c. Serum sodium level of 132 mmol/L d. Serum potassium level of 2.5 mmol/L

ANS: D Insulin activates the sodium-potassium ATPase pump, increasing the movement of potassium from the extracellular fluid into the intracellular fluid, resulting in hypokalemia. In hyperglycemia, hypokalemia can also result from excessive urine loss of potassium. The chloride level is normal. The calcium and sodium levels are slightly low, but this would not be related to hyperglycemia and insulin administration.

The UAP reports to you that a patient with type 1 diabetes has a question about exercise. What important points would you be sure to teach this patient? (Select all that apply.) 1. Exercise guidelines are based on blood glucose and urine ketone levels. 2. Be sure to test your blood glucose only after exercising. 3. You can exercise vigorously if your blood glucose is between 100 and 250 mg/dL. 4. Exercise will help resolve the presence of ketones in your urine. 5. A 5- to 10-minute warm-up and cool-down period should be included in your exercise.

Ans: 1, 3, 5 Guidelines for exercise are based on blood glucose and urine ketone levels. Patients should test blood glucose before, during, and after exercise to be sure that it is safe. When ketones are present in urine, the patient should not exercise because they indicate that current insulin levels are not adequate. Vigorous exercise is permitted in patients with type 1 diabetes if glucose levels are between 100 and 250 mg/dL. Warm-up and cool-down should be included in exercise to gradually increase and decrease the heart rate. Focus: Prioritization; QSEN: PCC, S; Concept: Patient Education

You are caring for a diabetic patient admitted with hypoglycemia that occurred at home. Which teaching points for treatment of hypoglycemia at home would you include in a teaching plan for the patient and family before discharge? (Select all that apply.) 1. Signs and symptoms of hypoglycemia include hunger, irritability, weakness, headache, and blood glucose less than 60 mg/dL. 2. Treat hypoglycemia with 4 to 8 g of carbohydrate such as glucose tablets or 1⁄4 cup of fruit juice. 3. Retest blood glucose in 30 minutes. 4. Repeat the carbohydrate treatment if the symptoms do not resolve. 5. Eat a small snack of carbohydrate and protein if the next meal is more than an hour away.

Ans: 1, 4, 5 The manifestations listed in option 1 are correct. The symptoms should be treated with carbohydrate, but 10 to 15 g (not 4 to 8 g). Glucose should be retested at 15 minutes; 30 minutes is too long to wait. Options 4 and 5 are correct. Focus: Prioritization; QSEN: PCC, S; Concept: Patient Education

While you are performing an admission assessment on a patient with type 2 diabetes, he tells you that he routinely drinks 3 beers a day. What is your priority follow-up question at this time? 1. "Do you have any days when you do not drink?" 2. "When during the day do you drink your beers?" 3. "Do you drink any other forms of alcohol?" 4. "Have you ever had a lipid profile completed?"

Ans: 2 Alcohol has the potential for causing alcohol-induced hypoglycemia. It is important to know when the patient drinks alcohol and to teach the patient to ingest it shortly after meals to prevent this complication. The other questions are important, but not urgent. The lipid profile question is important because alcohol can raise plasma triglycerides but is not as urgent as the potential for hypoglycemia. Focus: Prioritization; QSEN: S; Concept: Safety

In the emergency department during initial assessment of a newly-admitted patient with diabetes, the nurse discovers all of these findings. Which finding should be reported to the health care provider immediately? 1. Hammer toe of the left second metatarsophalangeal joint 2. Rapid respiratory rate with deep inspirations 3. Numbness and tingling bilaterally in the feet and hands 4. Decreased sensitivity and swelling of the abdomen

Ans: 2 Rapid, deep respirations (Kussmaul respirations) are symptomatic of diabetic ketoacidosis (DKA). Hammer toe, as well as numbness and tingling, are chronic complications associated with diabetes. Decreased sensitivity and swelling (lipohypertrophy) occurs at a site of repeated insulin injections, and treatment involves teaching the patient to rotate injection sites. Focus: Prioritization; QSEN: TC, S; Concept: Clinical Judgment

You are serving as preceptor to a nurse who has recently graduated and passed the RN licensure examination. The new nurse has only been on the unit for 2 days. Which patient should you assign to the new nurse? 1. 68-year-old with diabetes who is showing signs of hyperglycemia 2. 58-year-old with diabetes who has cellulitis of the left ankle 3. 49-year-old with diabetes who has just returned from the postanesthesia care unit after a below-knee amputation 4. 72-year-old with diabetes with DKA who is receiving IV insulin

Ans: 2 The new nurse is still on orientation to the unit. Appropriate patient assignments at this time include patients whose conditions are stable and not complex. Focus: Assignment; QSEN: TC, S; Concept: Leadership

A patient with diabetes has hot, dry skin; rapid and deep respirations; and a fruity odor to his breath. As charge nurse, you observe a newly-graduated RN performing all the following patient tasks. Which one requires that you intervene immediately? 1. Checking the patient's fingerstick glucose level 2. Encouraging the patient to drink orange juice 3. Checking the patient's order for sliding-scale insulin dosing 4. Assessing the patient's vital signs every 15 minutes

Ans: 2 The signs and symptoms the patient is exhibiting are consistent with hyperglycemia. The RN should not give the patient additional glucose. All of the other interventions are appropriate for this patient. The RN should also notify the provider at this time. Focus: Prioritization; QSEN: TC, S; Concept: Safety

You are the preceptor for a senior nursing student who will teach a diabetic patient about self-care during sick days. For which statement by the student must you intervene? 1. "When you are sick, be sure to monitor your blood glucose at least every 4 hours." 2. "Test your urine for ketones whenever your blood glucose level is less than 240 mg/dL." 3. "To prevent dehydration, drink 8 ounces of sugar-free liquid every hour while you are awake." 4. "Continue to eat your meals and snacks at the usual times."

Ans: 2 Urine ketone testing should be done whenever the patient's blood glucose is greater than 240 mg/dL. All of the other teaching points are appropriate "sick day rules." For dehydration, teaching should also include that if the patient's blood glucose is lower than her target range, she should drink fluids containing sugar. Focus: Supervision, delegation; QSEN: PCC, TC, S; Concept: Patient Education

Which actions can the school nurse delegate to UAPs who are working with a 7-year-old child with type 1 diabetes in an elementary school? (Select all that apply.) 1. Obtaining information about the child's usual insulin use from the parents 2. Administering oral glucose tablets when blood glucose level falls below 60 mg/dL 3. Teaching the child about what foods have high carbohydrate levels 4. Obtaining blood glucose readings using the child's blood glucose monitor 5. Reminding the child to have a snack after the physical education class

Ans: 2, 4, 5 National guidelines published by the American Diabetes Association (ADA) indicate that administration of emergency treatment for hypoglycemia, obtaining blood glucose readings, and reminding children are appropriate tasks for non-health care professional personnel such as teachers, paraprofessionals, and unlicensed health care personnel. Assessments and education require more specialized education and scope of practice and should be done by the school nurse. Focus: Delegation; QSEN: EBP, TC; Concept: Collaboration

An LPN/LVN's assessment of two diabetic patients reveals all of these findings. Which would you instruct the LPN/LVN to report immediately? 1. Fingerstick glucose reading of 185 mg/dL 2. Numbness and tingling in both feet 3. Profuse perspiration 4. Bunion on the left great toe

Ans: 3 Profuse perspiration is a symptom of hypoglycemia, a complication of diabetes that requires urgent treatment. A glucose level of 185 mg/dL will need coverage with sliding-scale insulin, but this is not urgent. Numbness and tingling, as well as bunions, are related to the chronic nature of diabetes and are not urgent problems. Focus: Prioritization; QSEN: TC, S; Concept: Communication

A patient has newly-diagnosed type 2 diabetes. Which action should you assign to an LPN/LVN instead of a UAP? 1. Measuring the patient's vital signs every shift 2. Checking the patient's glucose level before each meal 3. Administering subcutaneous insulin on a sliding scale as needed 4. Assisting the patient with morning care

Ans: 3 The UAP's scope of practice includes checking vital signs and assisting with morning care. UAPs with special training can check the patient's glucose level before meals. It is generally not within the UAP's scope of practice to administer medications, but this is within the scope of practice of the LPN/LVN. Focus: Assignment; QSEN: TC, S; Concept: Collaboration

The experienced UAP has been delegated to take vital signs and check fingerstick glucose on a diabetic patient who is postoperative. Which vital sign change would you instruct the UAP to report immediately? 1. Blood pressure increase from 132/80 mm Hg to 138/84 mm Hg 2. Temperature increase from 98.4° F (36.8° C) to 99° F (37.2° C) 3. Respiratory rate increase from 18 breaths/min to 22 breaths/min 4. Glucose increase from 190 mg/dL to 236 mg/dL

Ans: 4 An unexpected rise in blood glucose is associated with increased mortality and morbidity after surgical procedures. Current ADA guidelines recommend insulin protocols to maintain blood glucose levels between 140 and 180 mg/dL. Also, unexpected rises in blood glucose values may indicate wound infection. Focus: Delegation, supervision, prioritization; QSEN: PCC, TC, S; Concept: Communication

A patient with type 1 diabetes reports feeling dizzy. What should the nurse do first? 1. Check the patient's blood pressure. 2. Give the patient some orange juice. 3. Give the patient's morning dose of insulin. 4. Use a glucometer to check the patient's glucose level.

Ans: 4 Before orange juice or insulin is given, the patient's blood glucose level should be checked. Checking blood pressure is a good idea but is not the first action the nurse should take. Focus: Prioritization; QSEN: S; Concept: Clinical Judgment

A patient has newly-diagnosed type 2 diabetes. Which task should you delegate to a UAP? 1. Arranging a consult with the dietitian 2. Assessing the patient's insulin injection technique 3. Teaching the patient to use a glucometer to monitor glucose at home 4. Reminding the patient to check glucose level before each meal

Ans: 4 The UAP's role includes reminding patients about interventions that are already part of the plan of care. Arranging for a consult with the dietitian is appropriate for the unit clerk. Teaching and assessing require additional education and should be carried out by licensed nurses. Focus: Delegation, supervision, assignment; QSEN: TC; Concept: Collaboration

While working in the diabetes clinic, you obtain this information about an 8-year-old with type 1 diabetes. Which finding is most important to address when planning child and parent education? 1. Most recent hemoglobin A1c level of 7.8% 2. Many questions about diet choices from the parents 3. Child's participation in soccer practice after school 2 days a week 4. Morning preprandial glucose range of 55 to 70 mg/dL

Ans: 4 The low morning fasting blood glucose level indicates possible nocturnal hypoglycemia. Research indicates that it is important to avoid hypoglycemic episodes in pediatric patients because of the risk for permanent neurologic damage and adverse developmental outcomes. Although a lower hemoglobin A1c might be desirable, the upper limit for hemoglobin A1c levels ranges from 7.5% to 8.5% in pediatric patients. The parents' questions about diet and the child's activity level should also be addressed, but the most urgent consideration is education about the need to avoid hypoglycemia. Focus: Prioritization; QSEN: EBP, S; Concept: Patient Education

A UAP tells you that, while assisting with the morning care of a postoperative patient with type 2 diabetes who has been given insulin, the patient asked if she will always need to take insulin now. What is your priority for teaching the patient? 1. Explain to the patient that she is now considered to have type 1 diabetes. 2. Tell the patient to monitor fingerstick glucose level every 4 hours after discharge. 3. Teach the patient that a person with type 2 diabetes does not always need insulin. 4. Talk with the patient about the relationship between illness and increased glucose levels

Ans: 4 When a diabetic patient is ill or has surgery, glucose levels become elevated, and administration of insulin may be necessary. This is a temporary change that resolves with recovery from the illness or surgery. Option 3 is correct but does not explain why the patient may currently need insulin. The patient does not have type 1 diabetes, and fingerstick glucose checks are usually prescribed for before meals and at bedtime. Focus: Prioritization; QSEN: PCC; Concept: Patient Education

23.A patient with type 2 diabetes achieves adequate glycemic control through diet and exercise. Upon being admitted to the hospital for a cholecystectomy, however, the patient has required insulin injections on two occasions. The nurse would identify what likely cause for this short-term change in treatment? A) Alterations in bile metabolism and release have likely caused hyperglycemia. B) Stress has likely caused an increase in the patients blood sugar levels. C) The patient has likely overestimated her ability to control her diabetes using nonpharmacologic measures. D) The patients volatile fluid balance surrounding surgery has likely caused unstable blood sugars.

B Feedback: During periods of physiologic stress, such as surgery, blood glucose levels tend to increase, because levels of stress hormones (epinephrine, norepinephrine, glucagon, cortisol, and growth hormone) increase. The patients need for insulin is unrelated to the action of bile, the patients overestimation of previous blood sugar control, or fluid imbalance.

39.A diabetic patient calls the clinic complaining of having a flu bug. The nurse tells him to take his regular dose of insulin. What else should the nurse tell the patient? A) Make sure to stick to your normal diet. B) Try to eat small amounts of carbs, if possible. C) Ensure that you check your blood glucose every hour. D) For now, check your urine for ketones every 8 hours.

B Feedback: For prevention of DKA related to illness, the patient should attempt to consume frequent small portions of carbohydrates (including foods usually avoided, such as juices, regular sodas, and gelatin). Drinking fluids every hour is important to prevent dehydration. Blood glucose and urine ketones must be assessed every 3 to 4 hours.

33.A patient with a history of type 1 diabetes has just been admitted to the critical care unit (CCU) for diabetic ketoacidosis. The CCU nurse should prioritize what assessment during the patients initial phase of treatment? A) Monitoring the patient for dysrhythmias B) Maintaining and monitoring the patients fluid balance C) Assessing the patients level of consciousness D) Assessing the patient for signs and symptoms of venous thromboembolism

B Feedback: In addition to treating hyperglycemia, management of DKA is aimed at correcting dehydration, electrolyte loss, and acidosis before correcting the hyperglycemia with insulin. The nurse should monitor the patient for dysrhythmias, decreased LOC and VTE, but restoration and maintenance of fluid balance is the highest priority.

8.The nurse is discussing macrovascular complications of diabetes with a patient. The nurse would address what topic during this dialogue? A) The need for frequent eye examinations for patients with diabetes B) The fact that patients with diabetes have an elevated risk of myocardial infarction C) The relationship between kidney function and blood glucose levels D) The need to monitor urine for the presence of albumin

B Feedback: Myocardial infarction and stroke are considered macrovascular complications of diabetes, while the effects on vision and renal function are considered to be microvascular.

16.A nurse is caring for a patient with type 1 diabetes who is being discharged home tomorrow. What is the best way to assess the patients ability to prepare and self-administer insulin? A) Ask the patient to describe the process in detail. B) Observe the patient drawing up and administering the insulin. C) Provide a health education session reviewing the main points of insulin delivery. D) Review the patients first hemoglobin A1C result after discharge.

B Feedback: Nurses should assess the patients ability to perform diabetes related self-care as soon as possible during the hospitalization or office visit to determine whether the patient requires further diabetes teaching. While consulting a home care nurse is beneficial, an initial assessment should be performed during the hospitalization or office visit. Nurses should directly observe the patient performing the skills such as insulin preparation and infection, blood glucose monitoring, and foot care. Simply questioning the patient about these skills without actually observing performance of the skill is not sufficient. Further education does not guarantee learning.

5.A medical nurse is caring for a patient with type 1 diabetes. The patients medication administration record includes the administration of regular insulin three times daily. Knowing that the patients lunch tray will arrive at 11:45, when should the nurse administer the patients insulin? A) 10:45 B) 11:15 C) 11:45 D) 11:50

B Feedback: Regular insulin is usually administered 2030 min before a meal. Earlier administration creates a risk for hypoglycemia; later administration creates a risk for hyperglycemia.

36.A diabetes nurse is assessing a patients knowledge of self-care skills. What would be the most appropriate way for the educator to assess the patients knowledge of nutritional therapy in diabetes? A) Ask the patient to describe an optimally healthy meal. B) Ask the patient to keep a food diary and review it with the nurse. C) Ask the patients family what he typically eats. D) Ask the patient to describe a typical days food intake.

B Feedback: Reviewing the patients actual food intake is the most accurate method of gauging the patients diet.

6.A patient has just been diagnosed with type 2 diabetes. The physician has prescribed an oral antidiabetic agent that will inhibit the production of glucose by the liver and thereby aid in the control of blood glucose. What type of oral antidiabetic agent did the physician prescribe for this patient? A) A sulfonylurea B) A biguanide C) A thiazolidinedione D) An alpha glucosidase inhibitor

B Feedback: Sulfonylureas exert their primary action by directly stimulating the pancreas to secrete insulin and therefore require a functioning pancreas to be effective. Biguanides inhibit the production of glucose by the liver and are in used in type 2 diabetes to control blood glucose levels. Thiazolidinediones enhance insulin action at the receptor site without increasing insulin secretion from the beta cells of the pancreas. Alpha glucosidase inhibitors work by delaying the absorption of glucose in the intestinal system, resulting in a lower postprandial blood glucose level.

The nurse is taking a health history from a 29-yr-old pregnant patient at the first prenatal visit. The patient reports that she has no personal history of diabetes, but her mother has diabetes. Which action will the nurse plan to take? a. Teach the patient about administering regular insulin. b. Schedule the patient for a fasting blood glucose level. c. Teach about an increased risk for fetal problems with gestational diabetes. d. Schedule an oral glucose tolerance test for the twenty-fourth week of pregnancy.

B Patients at high risk for gestational diabetes should be screened for diabetes on the initial prenatal visit. An oral glucose tolerance test may also be used to check for diabetes, but it would be done before the twenty-fourth week. Teaching plans would depend on the outcome of a fasting blood glucose test and other tests.

17.An elderly patient comes to the clinic with her daughter. The patient is a diabetic and is concerned about foot care. The nurse goes over foot care with the patient and her daughter as the nurse realizes that foot care is extremely important. Why would the nurse feel that foot care is so important to this patient? A) An elderly patient with foot ulcers experiences severe foot pain due to the diabetic polyneuropathy. B) Avoiding foot ulcers may mean the difference between institutionalization and continued independent living. C) Hypoglycemia is linked with a risk for falls; this risk is elevated in older adults with diabetes. D) Oral antihyperglycemics have the possible adverse effect of decreased circulation to the lower extremities.

B Feedback: The nurse recognizes that providing information on the long-term complicationsespecially foot and eye problemsassociated with diabetes is important. Avoiding amputation through early detection of foot ulcers may mean the difference between institutionalization and continued independent living for the elderly person with diabetes. While the nurse recognizes that hypoglycemia is a dangerous situation and may lead to falls, hypoglycemia is not directly connected to the importance of foot care. Decrease in circulation is related to vascular changes and is not associated with drugs administered for diabetes.

An unresponsive patient with type 2 diabetes is brought to the emergency department and diagnosed with hyperosmolar hyperglycemic syndrome (HHS). The nurse will anticipate the need to a. give 50% dextrose. b. insert an IV catheter. c. initiate O2 by nasal cannula. d. administer glargine (Lantus) insulin.

B HHS is initially treated with large volumes of IV fluids to correct hypovolemia. Regular insulin is administered, not a long-acting insulin. There is no indication that the patient requires O2 . Dextrose solutions will increase the patient's blood glucose and would be contraindicated.

The nurse determines a need for additional instruction when the patient with newly diagnosed type 1 diabetes says which of the following? a. "I will need a bedtime snack because I take an evening dose of NPH insulin." b. "I can choose any foods, as long as I use enough insulin to cover the calories." c. "I can have an occasional beverage with alcohol if I include it in my meal plan." d. "I will eat something at meal times to prevent hypoglycemia, even if I am not hungry."

B Most patients with type 1 diabetes need to plan diet choices very carefully. Patients who are using intensified insulin therapy have considerable flexibility in diet choices but still should restrict dietary intake of items such as fat, protein, and alcohol. The other patient statements are correct and indicate good understanding of the diet instruction.

Which action should the nurse take after a patient treated with intramuscular glucagon for hypoglycemia regains consciousness? a. Assess the patient for symptoms of hyperglycemia. b. Give the patient a snack of peanut butter and crackers. c. Have the patient drink a glass of orange juice or nonfat milk. d. Administer a continuous infusion of 5% dextrose for 24 hours.

B Rebound hypoglycemia can occur after glucagon administration, but having a meal containing complex carbohydrates plus protein and fat will help prevent hypoglycemia. Orange juice and nonfat milk will elevate blood glucose rapidly, but the cheese and crackers will stabilize blood glucose. Administration of IV glucose might be used in patients who were unable to take in nutrition orally. The patient should be assessed for symptoms of hypoglycemia after glucagon administration.

Which laboratory value reported to the nurse by the unlicensed assistive personnel (UAP) indicates an urgent need for the nurse's assessment of the patient? a. Bedtime glucose of 140 mg/dL b. Noon blood glucose of 52 mg/dL c. Fasting blood glucose of 130 mg/dL d. 2-hr postprandial glucose of 220 mg/dL

B The nurse should assess the patient with a blood glucose level of 52 mg/dL for symptoms of hypoglycemia and give the patient a carbohydrate-containing beverage such as orange juice. The other values are within an acceptable range or not immediately dangerous for a patient with diabetes.

13.A diabetes educator is teaching a patient about type 2 diabetes. The educator recognizes that the patient understands the primary treatment for type 2 diabetes when the patient states what? A) I read that a pancreas transplant will provide a cure for my diabetes. B) I will take my oral antidiabetic agents when my morning blood sugar is high. C) I will make sure to follow the weight loss plan designed by the dietitian. D) I will make sure I call the diabetes educator when I have questions about my insulin.

C Feedback: Insulin resistance is associated with obesity; thus the primary treatment of type 2 diabetes is weight loss. Oral antidiabetic agents may be added if diet and exercise are not successful in controlling blood glucose levels. If maximum doses of a single category of oral agents fail to reduce glucose levels to satisfactory levels, additional oral agents may be used. Some patients may require insulin on an ongoing basis or on a temporary basis during times of acute psychological stress, but it is not the central component of type 2 treatment. Pancreas transplantation is associated with type 1 diabetes.

1.A patient with type 1 diabetes has told the nurse that his most recent urine test for ketones was positive. What is the nurses most plausible conclusion based on this assessment finding? A) The patient should withhold his next scheduled dose of insulin. B) The patient should promptly eat some protein and carbohydrates. C) The patients insulin levels are inadequate. D) The patient would benefit from a dose of metformin (Glucophage).

C Feedback: Ketones in the urine signal that there is a deficiency of insulin and that control of type 1 diabetes is deteriorating. Withholding insulin or eating food would exacerbate the patients ketonuria. Metformin will not cause short-term resolution of hyperglycemia.

24.A physician has explained to a patient that he has developed diabetic neuropathy in his right foot. Later that day, the patient asks the nurse what causes diabetic neuropathy. What would be the nurses best response? A) Research has shown that diabetic neuropathy is caused by fluctuations in blood sugar that have gone on for years. B) The cause is not known for sure but it is thought to have something to do with ketoacidosis. C) The cause is not known for sure but it is thought to involve elevated blood glucose levels over a period of years. D) Research has shown that diabetic neuropathy is caused by a combination of elevated glucose levels and elevated ketone levels.

C Feedback: The etiology of neuropathy may involve elevated blood glucose levels over a period of years. High blood sugars (rather than fluctuations or variations in blood sugars) are thought to be responsible. Ketones and ketoacidosis are not direct causes of neuropathies.

19.Which of the following patients with type 1 diabetes is most likely to experience adequate glucose control? A) A patient who skips breakfast when his glucose reading is greater than 220 mg/dL B) A patient who never deviates from her prescribed dose of insulin C) A patient who adheres closely to a meal plan and meal schedule D) A patient who eliminates carbohydrates from his daily intake

C Feedback: The therapeutic goal for diabetes management is to achieve normal blood glucose levels without hypoglycemia. Therefore, diabetes management involves constant assessment and modification of the treatment plan by health professionals and daily adjustments in therapy (possibly including insulin) by patients. For patients who require insulin to help control blood glucose levels, maintaining consistency in the amount of calories and carbohydrates ingested at meals is essential. In addition, consistency in the approximate time intervals between meals, and the snacks, help maintain overall glucose control. Skipping meals is never advisable for person with type 1 diabetes.

Which statement by a nurse to a patient newly diagnosed with type 2 diabetes is correct? a. Insulin is not used to control blood glucose in patients with type 2 diabetes. b. Complications of type 2 diabetes are less serious than those of type 1 diabetes. c. Changes in diet and exercise may control blood glucose levels in type 2 diabetes. d. Type 2 diabetes is usually diagnosed when the patient is admitted with a hyperglycemic coma.

C For some patients with type 2 diabetes, changes in lifestyle are sufficient to achieve blood glucose control. Insulin is frequently used for type 2 diabetes, complications are equally severe as for type 1 diabetes, and type 2 diabetes is usually diagnosed with routine laboratory testing or after a patient develops complications such as frequent yeast infections.

When a patient who takes metformin (Glucophage) to manage type 2 diabetes develops an allergic rash from an unknown cause, the health care provider prescribes prednisone. The nurse will anticipate that the patient may a. need a diet higher in calories while receiving prednisone. b. develop acute hypoglycemia while taking the prednisone. c. require administration of insulin while taking prednisone. d. have rashes caused by metformin-prednisone interactions.

C Glucose levels increase when patients are taking corticosteroids, and insulin may be required to control blood glucose. Hypoglycemia is not a side effect of prednisone. Rashes are not an adverse effect caused by taking metformin and prednisone simultaneously. The patient may have an increased appetite when taking prednisone but will not need a diet that is higher in calories.

The health care provider suspects the Somogyi effect in a 50-yr-old patient whose 6:00 AM blood glucose is 230 mg/dL. Which action will the nurse teach the patient to take? a. Avoid snacking at bedtime. b. Increase the rapid-acting insulin dose. c. Check the blood glucose during the night d. Administer a larger dose of long-acting insulin.

C If the Somogyi effect is causing the patient's increased morning glucose level, the patient will experience hypoglycemia between 2:00 and 4:00 AM. The dose of insulin will be reduced, rather than increased. A bedtime snack is used to prevent hypoglycemic episodes during the night.

A 26-yr-old female with type 1 diabetes develops a sore throat and runny nose after caring for her sick toddler. The patient calls the clinic for advice about her symptoms and a blood glucose level of 210 mg/dL despite taking her usual glargine (Lantus) and lispro (Humalog) insulin. The nurse advises the patient to a. use only the lispro insulin until the symptoms are resolved. b. limit intake of calories until the glucose is less than 120 mg/dL. c. monitor blood glucose every 4 hours and notify the clinic if it continues to rise. d. decrease intake of carbohydrates until glycosylated hemoglobin is less than 7%.

C Infection and other stressors increase blood glucose levels and the patient will need to test blood glucose frequently, treat elevations appropriately with lispro insulin, and call the health care provider if glucose levels continue to be elevated. Discontinuing the glargine will contribute to hyperglycemia and may lead to diabetic ketoacidosis (DKA). Decreasing carbohydrate or caloric intake is not appropriate because the patient will need more calories when ill. Glycosylated hemoglobin testing is not used to evaluate short-term alterations in blood glucose.

A patient with diabetes rides a bicycle to and from work every day. Which site should the nurse teach the patient to use to administer the morning insulin? a. thigh. c. abdomen. b. buttock. d. upper arm.

C Patients should be taught not to administer insulin into a site that will be exercised because exercise will increase the rate of absorption. The thigh, buttock, and arm are all exercised by riding a bicycle.

A hospitalized diabetic patient received 38 U of NPH insulin at 7:00 AM. At 1:00 PM, the patient has been away from the nursing unit for 2 hours, missing the lunch delivery while awaiting a chest x-ray. To prevent hypoglycemia, the best action by the nurse is to a. save the lunch tray for the patient's later return to the unit. b. ask that diagnostic testing area staff to start a 5% dextrose IV. c. send a glass of milk or orange juice to the patient in the diagnostic testing area. d. request that if testing is further delayed, the patient be returned to the unit to eat.

D Consistency for mealtimes assists with regulation of blood glucose, so the best option is for the patient to have lunch at the usual time. Waiting to eat until after the procedure is likely to cause hypoglycemia. Administration of an IV solution is unnecessarily invasive for the patient. A glass of milk or juice will keep the patient from becoming hypoglycemic but will cause a rapid rise in blood glucose because of the rapid absorption of the simple carbohydrate in these items.

38.A nurse is assessing a patient who has diabetes for the presence of peripheral neuropathy. The nurse should question the patient about what sign or symptom that would suggest the possible development of peripheral neuropathy? A) Persistently cold feet B) Pain that does not respond to analgesia C) Acute pain, unrelieved by rest D) The presence of a tingling sensation

D Feedback: Although approximately half of patients with diabetic neuropathy do not have symptoms, initial symptoms may include paresthesias (prickling, tingling, or heightened sensation) and burning sensations (especially at night). Cold and intense pain are atypical early signs of this complication.

31.A patient has received a diagnosis of type 2 diabetes. The diabetes nurse has made contact with the patient and will implement a program of health education. What is the nurses priority action? A) Ensure that the patient understands the basic pathophysiology of diabetes. B) Identify the patients body mass index. C) Teach the patient survival skills for diabetes. D) Assess the patients readiness to learn.

D Feedback: Before initiating diabetes education, the nurse assesses the patients (and familys) readiness to learn. This must precede other physiologic assessments (such as BMI) and providing health education.

14.A diabetes nurse educator is presenting the American Diabetes Association (ADA) recommendations for levels of caloric intake. What do the ADAs recommendations include? A) 10% of calories from carbohydrates, 50% from fat, and the remaining 40% from protein B) 10% to 20% of calories from carbohydrates, 20% to 30% from fat, and the remaining 50% to 60% from protein C) 20% to 30% of calories from carbohydrates, 50% to 60% from fat, and the remaining 10% to 20% from protein D) 50% to 60% of calories from carbohydrates, 20% to 30% from fat, and the remaining 10% to 20% from protein

D Feedback: Currently, the ADA and the Academy of Nutrition and Dietetics (formerly the American Dietetic Association) recommend that for all levels of caloric intake, 50% to 60% of calories come from carbohydrates, 20% to 30% from fat, and the remaining 10% to 20% from protein.

21.A medical nurse is aware of the need to screen specific patients for their risk of hyperglycemic hyperosmolar syndrome (HHS). In what patient population does hyperosmolar nonketotic syndrome most often occur? A) Patients who are obese and who have no known history of diabetes B) Patients with type 1 diabetes and poor dietary control C) Adolescents with type 2 diabetes and sporadic use of antihyperglycemics D) Middle-aged or older people with either type 2 diabetes or no known history of diabetes

D Feedback: HHS occurs most often in older people (50 to 70 years of age) who have no known history of diabetes or who have type 2 diabetes.

29.A nurse is teaching basic survival skills to a patient newly diagnosed with type 1 diabetes. What topic should the nurse address? A) Signs and symptoms of diabetic nephropathy B) Management of diabetic ketoacidosis C) Effects of surgery and pregnancy on blood sugar levels D) Recognition of hypoglycemia and hyperglycemia

D Feedback: It is imperative that newly diagnosed patients know the signs and symptoms and management of hypo- and hyperglycemia. The other listed topics are valid points for education, but are not components of the patients immediate survival skills following a new diagnosis.

15.An older adult patient with type 2 diabetes is brought to the emergency department by his daughter. The patient is found to have a blood glucose level of 623 mg/dL. The patients daughter reports that the patient recently had a gastrointestinal virus and has been confused for the last 3 hours. The diagnosis of hyperglycemic hyperosmolar syndrome (HHS) is made. What nursing action would be a priority? A) Administration of antihypertensive medications B) Administering sodium bicarbonate intravenously C) Reversing acidosis by administering insulin D) Fluid and electrolyte replacement

D Feedback: The overall approach to HHS includes fluid replacement, correction of electrolyte imbalances, and insulin administration. Antihypertensive medications are not indicated, as hypotension generally accompanies HHS due to dehydration. Sodium bicarbonate is not administered to patients with HHS, as their plasma bicarbonate level is usually normal. Insulin administration plays a less important role in the treatment of HHS because it is not needed for reversal of acidosis, as in diabetic ketoacidosis (DKA).

11.A newly admitted patient with type 1 diabetes asks the nurse what caused her diabetes. When the nurse is explaining to the patient the etiology of type 1 diabetes, what process should the nurse describe? A) The tissues in your body are resistant to the action of insulin, making the glucose levels in your blood increase. B) Damage to your pancreas causes an increase in the amount of glucose that it releases, and there is not enough insulin to control it. C) The amount of glucose that your body makes overwhelms your pancreas and decreases your production of insulin. D) Destruction of special cells in the pancreas causes a decrease in insulin production. Glucose levels rise because insulin normally breaks it down.

D Feedback: Type 1 diabetes is characterized by the destruction of pancreatic beta cells, resulting in decreased insulin production, unchecked glucose production by the liver, and fasting hyperglycemia. Also, glucose derived from food cannot be stored in the liver and remains circulating in the blood, which leads to postprandial hyperglycemia. Type 2 diabetes involves insulin resistance and impaired insulin secretion. The body does not make glucose.

The nurse is interviewing a new patient with diabetes who takes rosiglitazone (Avandia). Which information would the nurse anticipate resulting in the health care provider discontinuing the medication? a. The patient's blood pressure is 154/92. b. The patient's blood glucose is 86 mg/dL. c. The patient reports a history of emphysema. d. The patient has chest pressure when walking.

D Rosiglitazone can cause myocardial ischemia. The nurse should immediately notify the health care provider and expect orders to discontinue the medication. A blood glucose level of 86 mg/dL indicates a positive effect from the medication. Hypertension and a history of emphysema do not contraindicate this medication.

A 28-yr-old male patient with type 1 diabetes reports how he manages his exercise and glucose control. Which behavior indicates that the nurse should implement additional teaching? a. The patient always carries hard candies when engaging in exercise. b. The patient goes for a vigorous walk when his glucose is 200 mg/dL. c. The patient has a peanut butter sandwich before going for a bicycle ride. d. The patient increases daily exercise when ketones are present in the urine.

D When the patient is ketotic, exercise may result in an increase in blood glucose level. Patients with type 1 diabetes should be taught to avoid exercise when ketosis is present. The other statements are correct.


Conjuntos de estudio relacionados

TLB-Chapter 39: Oxygenation and Perfusion

View Set

Nursing iclicker/ ati/module questions

View Set

Medical Terminology Module 10 Vocab- Nervous System and Brain

View Set

BIO 277 01- Exam 1 (Unit 1 objectives)

View Set

AP Psychology Abnormal Module 68

View Set

Chapter 7.3 End of Chapter Assessment

View Set

Google Apps for Education Chrome Browser

View Set

MGT 3900 McGraw-Hill Chapter 2, MGT 3900 McGraw-Hill Chapter 3

View Set

Ch 64: Arthritis and Connective Tissue Diseases

View Set

Intro to Criminal Justice 2381 Ch.7/8 Test

View Set